Number properties - prime factors

This topic has expert replies
User avatar
Legendary Member
Posts: 626
Joined: Fri Dec 23, 2011 2:50 am
Location: Ahmedabad
Thanked: 31 times
Followed by:10 members

by ronnie1985 » Fri Apr 06, 2012 9:35 am
Given: 2< y < 100 and y = perfect square
S1. Exactly two factors.
Writing down all the squares < 100 we get that 36 is the only square which satisfies the given condition,, Sufficient
S2. Even Number, Many even squares are there between given limits, Not Sufficient

(A) is answer.
Follow your passion, Success as perceived by others shall follow you

User avatar
Legendary Member
Posts: 1665
Joined: Thu Nov 03, 2011 7:04 pm
Thanked: 165 times
Followed by:70 members

by karthikpandian19 » Tue Jun 26, 2012 5:10 pm
This would be a typical word translation problem for which we will fall trap for....nice problem
Regards,
Karthik
The source of the questions that i post from JUNE 2013 is from KNEWTON

---If you find my post useful, click "Thank" :) :)---
---Never stop until cracking GMAT---

Senior | Next Rank: 100 Posts
Posts: 57
Joined: Sat May 15, 2010 5:46 am
Thanked: 2 times

by anujan007 » Mon Jul 23, 2012 1:20 pm
I also assumed that two factors would be 2 different ones. Do not think that such a problem will appear on the actual GMAT.
My attempt to capture my B-School Journey in a Blog : tranquilnomadgmat.blocked

There are no shortcuts to any place worth going.

Master | Next Rank: 500 Posts
Posts: 171
Joined: Tue Jan 08, 2013 7:24 am
Thanked: 1 times

by rajeshsinghgmat » Wed Apr 24, 2013 5:29 am
C the Answer.

User avatar
Master | Next Rank: 500 Posts
Posts: 104
Joined: Fri Oct 07, 2011 10:23 pm
Thanked: 36 times
Followed by:4 members

by fcabanski » Wed Apr 24, 2013 12:27 pm
If the problem meant distinct prime factors, it would use the word distinct.

User avatar
Master | Next Rank: 500 Posts
Posts: 234
Joined: Tue Jul 16, 2013 9:00 am
Location: West Virginia
Thanked: 9 times

by Java_85 » Fri Nov 15, 2013 1:54 pm
+1 C

Senior | Next Rank: 100 Posts
Posts: 44
Joined: Fri Apr 20, 2012 1:02 pm

by justharsha » Tue Dec 10, 2013 5:54 pm
IMO is A

Taking one fact at a time.
Fact 1 (from the Q): 2 < y < 100
Fact 2 (from the Q): y is square of a number

just from this info the possible answers are 1, 4, 9, 16, 25, 49, 64, 81

Fact 3 (from Statement 1): y has exactly 2 prime factors
from the numbers listed about only 36 uniquely satisfies the condition. Hence Sufficient

Hence the only possible answers now are A and D

Statement 2 gives multiple choices. It could be 4, 16 or 64 and hence insufficient

The correct answer is A

OA?

User avatar
Master | Next Rank: 500 Posts
Posts: 164
Joined: Sat Sep 20, 2014 10:26 pm
Thanked: 1 times

by jaspreetsra » Sun Jan 04, 2015 1:11 am
If y is an integer such that 2 < y < 100 and if y is also the square of an integer, what is the value of y?

1. y has exactly two prime factors.
2. y is even.
IMO: A
Reason:
Question Stem: Y is between 2 and 100 and also the square of an integer.
So, y may be one of 4,9,16,25,36,49,64,81

St. 1 y has exactly two prime factors. (36 has two prime nos: 2 and 3) Sufficient
Sr. 2 y is even.(Y may be 4, 16, 36, and 64) Not sufficient
Hard work brings success!

Legendary Member
Posts: 518
Joined: Tue May 12, 2015 8:25 pm
Thanked: 10 times

by nikhilgmat31 » Mon Sep 07, 2015 12:14 am
Can anyone post the OA for this question.

I believe it should be C for number y=4

but we can't deny y=36 with 2 prime factors 2 & 3 and Answer to be A

User avatar
Elite Legendary Member
Posts: 3991
Joined: Fri Jul 24, 2015 2:28 am
Location: Las Vegas, USA
Thanked: 19 times
Followed by:37 members

by Max@Math Revolution » Tue Sep 08, 2015 6:13 am
Forget conventional ways of solving math questions. In DS, Variable approach is the easiest and quickest way to find the answer without actually solving the problem. Remember equal number of variables and equations ensures a solution.



If y is an integer such that 2 < y < 100 and if y is also the square of an integer, what is the value of y ?

(1) y has exactly two prime factors.

(2) y is even.

In the original condition, there is 1 variable (y) and thus we need 1 equation to match the number of variable and equation. Since there is 1 each in 1) and 2), D is likely the answer.

In case of 1), y=4,9,25,49 and the answer is not unique. Therefore it is not sufficient.
In case of 2), y=4,16, 36, 64 the answer is not unique. Therefore it is not sufficient.
Using both 1) & 2) together, y=4 is unique. Therefore it is sufficient and the answer is C.


If you know our own innovative logics to find the answer, you don't need to actually solve the problem.
www.mathrevolution.com
- The one-and-only World's First Variable Approach for DS and IVY Approach for PS that allow anyone to easily solve GMAT math questions.

- The easy-to-use solutions. Math skills are totally irrelevant. Forget conventional ways of solving math questions.

- The most effective time management for GMAT math to date allowing you to solve 37 questions with 10 minutes to spare

- Hitting a score of 45 is very easy and points and 49-51 is also doable.

- Unlimited Access to over 120 free video lessons at https://www.mathrevolution.com/gmat/lesson

- Our advertising video at https://www.youtube.com/watch?v=R_Fki3_2vO8

Master | Next Rank: 500 Posts
Posts: 418
Joined: Sun Jul 04, 2010 12:48 pm
Thanked: 6 times
Followed by:3 members

by gmatdriller » Tue Sep 08, 2015 11:18 pm
4 = 2 x 2 (factors of 4 = 1, 2, & 4)...only 1 prime factor(2)
9 = 3 x 3 (factors of 9 = 1, 3, & 9)...only 1 prime factor(3)
16 = 4 x 4(factors of 16 = 1,2,4,8,& 16)...only 1 prime factor(2)
25 = 5 x 5 (factors of 25 = 1,5, & 25)....only 1 prime factor(5)
36 = 4 x 9 (factors of 36= 1,2,3,4,6,8,12,18,& 36) 2 prime factors (2 and 3)
...... until we get to 81, which also 1 prime factor (3)

That 4 = 2 x 2 does not imply 4 has 2 prime factors. Else, how many factors does it have?
In this case, I feel A should be the answer, right?

Pls what is the OA and explanations.

GMAT Instructor
Posts: 2630
Joined: Wed Sep 12, 2012 3:32 pm
Location: East Bay all the way
Thanked: 625 times
Followed by:119 members
GMAT Score:780

by Matt@VeritasPrep » Fri Sep 25, 2015 11:48 am
gmatdriller, I'm with you: if the question doesn't specify, then "two prime factors" (for me, at least) should be taken as two distinct prime factors. (For instance, 4 has three factors: 1, 2, and 4. One of those is prime.)

Newbie | Next Rank: 10 Posts
Posts: 4
Joined: Wed Aug 15, 2012 7:03 am
Location: Los Angeles

by dorschrm » Fri May 05, 2017 8:36 am
This question showed up in my e-mail inbox as a "Question of the Day."

To the people who choose the Question of the Day: questions like these, with clear ambiguity, aren't helping anyone. This is a recurring issue in the questions that have been selected and make their way to my inbox. I hope this can be cleaned up.

I appreciate the viewpoints of the regular experts on this site, they will keep me coming back regardless.